【圖論學習筆記二】雙計數(Double Counting)

雙計數狹義上講,對於一個集合運用兩種不同的方式,得到精確表達式等式結果或者不等式的結果。

 握手定理

對於圖G=(V,E),有\sum_{v\in V}d(v)=2|E|,即所有點的度爲邊的二倍。

推論:圖中奇度點數目爲偶數。

可以用關聯矩陣角度看握手定理(自己可以畫一個簡單的圖,寫出它的關聯矩陣);對關聯矩陣中“1”的個數進行計數,即對集合{(v,e)|v∈e},可以從行計數也可以從列計數。

握手定理對超圖也適用,例點集合V={1,2,3,4,5},邊集合\beta={{1,2},{2,3,4},{2,4,5}},\sum_{v\in V}d(v)=\sum_{B \in \beta } |B|

 等式證明

1.\binom{n}{k}=\frac{n}{k}\binom{n-1}{k-1}

證明:

用集合的雙計數進行證明。原式=k\binom{n}{k}=n\binom{n-1}{k-1},此時可解釋爲從n個元素的集合中,挑出滿足二元組的元素{(x,A)|x∈A,|A|=k},以不同元素開頭的k元集合的數目,這時可從集合兩個角度來看問題。1)可以先從n元集合中選出k元集合,再從k個元素中挑出開頭元素,即k\binom{n}{k};2)還可以先選定開頭元素,有n種選擇,再從n-1個元素中選擇k-1個元素。即n\binom{n-1}{k-1}。兩者是從不同角度進行計數,則等式成立。

從關聯矩陣的角度解釋,寫出|M|_{|x|*|A|}的矩陣(x在A上記爲1),分別對行和列計數:

M=\begin{bmatrix} & & \\ & 0-1 & \\ & & \end{bmatrix},從行計數來看,例如包含1的A_j的數目有\binom{n-1}{k-1},共有n個元素,則從行計數共有n\binom{n-1}{k-1}個1;從列計數來看,A_j列的1一定是k個,共有\binom{n}{k}個列。則從列計數共有k\binom{n}{k}個1。

 2.\sum_{k=1}^{n}k\binom{n}{k} =n2^{n-1}

證明:

上面的式子是對固定k的集合計數,而這個式子對k不做固定,k從1-n變化\sum_{k=1}^{n}k\binom{n}{k} =n\sum_{k=1}^{n}\binom{n-1}{k-1}=n2^{n-1}

3.\binom{n}{k}\binom{k}{l}=\binom{n}{l}\binom{n-l}{k-l},其中0\leq l\leq k\leq n

證明:

從集合角度看,可以看作這樣的二元集合計數{(A,B)||A|=l,|B|=k,且A\subseteq B}。先從n元集合中選出k元集合,再從k元集合中選出l元集合,即等式左邊;從n元集合選出l元集合,但要保證l元集合在k元集合裏,那麼包含l元集合的k元集合剩餘元素從n中選出填入,即等式右邊。

這裏也可以用關聯矩陣進行證明。(以上述二元組計數原則寫出關聯矩陣)

4.三計數,在超圖中,有\sum_{x\in X}d^2(x)=\sum_{A\in F} \sum_{x\in A}d(x)=\sum_{A\in F} \sum_{B\in F}|A\cap B|

對於集合(X,F),X爲點集合,F爲超邊集合,上式是對這樣的集合進行計數,{(x,A,B)|A,B∈F,x∈A,x∈B}。第一個式子x在全集X上,A,B集合均包含點x;第二個式子先選出在A的x,再使得x在B中;第三個式子先選出A和B集合,使得x在A和B上,那麼就是在A和B的交集上。

5.圖蘭定理T(n,k,l)\geq \binom{n}{l}/\binom{k}{l}

圖蘭數T(n,k,l) (I<=k<=n)是n元集合X的l元子集的最小值,使得X的每個k元子集至少包含一個這樣的l元集合。

設F爲滿足條件的l元集合,記F={A1,A2,...},此時可用關聯矩陣M_{|Ai|*|Bi|}表示,Ai爲滿足條件的l元集合,Bi爲\binom{n}{k}個k元集合,若Ai在Bi上,則爲1,就可以得到一個0-1矩陣。行計數:對於某一個l元集合,有\binom{n-l}{k-l}個k元集合包含它,共有|F|個,行計數1的個數爲|F|\binom{n-l}{k-l};列計數來看,每個Bi必包含一個Ai,則每一列至少有一個1,可以得到|F|\binom{n-l}{k-l}\geq \binom{n}{k},應用前面的3等式,可得T(n,k,l)=|F|\geq \binom{n}{l}/\binom{k}{l}

6.H-free圖;

設H是一個固定的圖形。如果一個圖不包含H作爲子圖,那麼它就是無H的。圖論中的一個典型問題是:一個有n個頂點的無H圖可以最多有多少條邊?

定理:如果圖G=(V,E)中不包含4個點的圈,那麼有|E|\leq \left \lfloor n/4(1+\sqrt{4n-3}) \right \rfloor

證明:

令點集V={1,2,...,n},用於雙計數的集合爲S={(u,{v,w})|uyuv,w都鄰接,且v≠w},固定u,那麼v和w只能在度爲d(u)的點中選取,即\binom{d(u)}{2},那麼就有\sum _{u\in V}\binom{d(u)}{2}=|S|;固定v和w,最多隻有一個點可以和它們都關聯,那麼有\binom{n}{2}\geq |S|,那麼,\sum _{u\in V}d^2(u)\leq \sum _{u\in V}d(u)+n(n-1),由柯西—施瓦茨不等式得,n\sum_{u\in V}d^2(n)\geq (\sum_{u\in V}d(n)*1)^2,代入上式,\frac{(\sum_{u\in V} d(u))^2}{n}\leq \sum_{u\in V}d(u)+n)n-1,運用握手定理得4|E|^2\leq n2|E|+n^2(n-1),求解n可得上述結論。

柯西—施瓦茨不等式,|(\alpha ,\beta )|\leq ||\alpha ||*||\beta ||,上述得不等式中對應的分別是(d(u_1),d(u_2)....d(u_n)),(1,1,...1)。

7.假設我們有兩個有限集R和C和一個子集S\subseteqRxC。無論何時(p,g)∈s 那麼認爲p和q是關聯的。設r_p表示p固定,與p關聯的元素數目;c_p表示q固定,與q關聯的元素數目。那麼有\sum_{p\in R}r_p=|S|=\sum_{q\in C}c_q

證明:

可以假設M_{|p|x|q|}的關聯矩陣使用雙計數進行證明,矩陣中若pi和qj相關聯,aij則置爲1,否則爲0,那麼|S|就是矩陣M中全部1的個數,等式的第一項可從行計數,最後一項可從列計數角度統計矩陣中1的個數。

設R=C={1,2,...,n},集合S={(i,j)|i可以整除j},t(j)表示j的因子的數目,如j=4,因子有1,2,4,那麼t(4)=3。對於\overline{t(j)}=\frac{1}{n}\sum_{j=1}^{n}t(j)有多大?

i\j 1 2 3 4 5 6 7 8
1 1 1 1 1 1 1 1 1
2   1   1   1   1
3     1     1    
4       1       1
5         1      
6           1    
7             1  
8               1

列計數轉化爲行計數。\overline{t(j)}=\frac{1}{n}\sum_{j=1}^{n}t(j)=\frac{1}{n}\sum_{i=1}^{n}\left \lfloor n/i \right \rfloor,有\frac{1}{n}\sum_{i=1}^{n}(\frac{n}{j}-1)<\frac{1}{n}\sum_{i=1}^{n}\left \lfloor n/i \right \rfloor\leq \frac{1}{n}\sum_{i=1}^{n}\frac{n}{i}=\sum_{i=1}^{n}\frac{1}{i},那麼\sum_{i=1}^{n}\frac{1}{i}-1<t(j)\leq lnn,(lnn<\sum_{i=1}^{n}\frac{1}{i}<lnn+1,具體證明可參考求和的積分近似),|\overline{t(j)}-lnn|<1

8.\binom{n}{2}=\binom{k}{2}+\binom{n-k}{2}+k(n-k)

證明:

 對於一個完全圖,共有\binom{n}{2}條邊;將完全圖劃分爲兩部分,一部分有k個點,則另一部分有n-k個點,k個點之間有\binom{k}{2}條邊,n-k個點之間有\binom{n-k}{2}條邊,k個點與n-k條邊之間有k(n-k)條邊。雙計數法。

可推廣:分爲S部分,\sum_{i=1}^{S}n_i=n,\binom{n}{2}=\sum_{i=1}^{S}\binom{n_i}{2}+\sum_{i<j}n_in_j

9.Zarankiewicz's problem

對於一個nxn的0-1矩陣,如果不存在axb的全1子矩陣,那麼這個nxn的矩陣最多有多少個1?

用二部圖重新表述這個問題。一個部分大小爲n的二部圖是一個三重G=(V1,V2,E),其中V1,V2是頂點的不相交n元集合,E\in V1\times V2是邊的集合。

令Ka(n)爲最小整數k邊,使得任意大小爲n且邊數大於k的二部圖至少包含一個axa clique。對於任意的自然數n和a,有K_a(n)\leq (a-1)^{1/a}n^{2-1/a}+(a-1)n

定義S={(x,A)|x∈V1,A∈V2,|A|=a,且x與A中的每個元素都有邊相連}

固定x:那麼從與x相連的元素中取A,假定x的neighbor爲d(x),那麼A有\sum_{x\in V_1}\binom{d(x)}{a}種選法,且其等於|S|;

固定A:從n中選出a元集合,與之對應相連的x最多有(a-1)個,否則就會出現axa clique,即\binom{n}{a}(a-1)

jensen不等式:對於凸函數有,\sum_{i=1}^{S}\lambda_1=1,\sum_{i=1}^{S}\lambda_if(x_i)\geq f(\sum_{i=1}^{S}\lambda_ix_i)

f(x)=\binom{x}{a},x_i=d(x_i);\frac{1}{n}\sum_{x\in V_1}\binom{d(x)}{a}\geq \binom{\frac{1}{n}\sum_{x\in V_1}d(x)}{a}\sum_{x\in V_1}\binom{d(x)}{a}\geq n\binom{\frac{1}{n}\sum_{x\in V_1}d(x)}{a}=n\binom{\frac{|E|}{n}}{a}(因爲爲二部圖,則degree爲一倍邊數),且有|S|≤\binom{n}{a}(a-1),即n\binom{\frac{|E|}{n}}{a}\leq \binom{n}{a}(a-1),經過放大縮小有n(\frac{|E|}{n}-(a-1))^a<\frac{n(|E|/n)(|E|/n-1)...(|E|/n-(a-1))}{a!}=n\binom{\frac{|E|}{n}}{a}\binom{n}{a}(a-1)=\frac{n(n-1)...(n-(a-1))}{a!}(a-1)<\frac{n^a}{a!}(a-1),兩邊同時開a次方即可解的。

Jensen不等式可以對求和項數r做數學歸納法證明‘

Ka(n)的下界可以用概率方法求得。

10.斯波納引理(Sperner Lemma)

假設某個頂點爲V1、V2、V3的“大”三角形被三角化了(也就是說,被分解成有限數量的“小”三角形,這些“小”三角形每條邊都能拼接在一起)。假設三角化中的頂點從集合{1,2,3}中獲取顏色,使得Vi接收顏色i(對於每個i),並且沿着Vi到Vi的邊的頂點只用i和j的顏色,而內部頂點用1、2或3的顏色任意着色。那麼在三角測量中一定有一個小的“三色”三角形。

證明:

假定大三角形外部有一點A,每一個小三角形中心都有一個頂點O,若小三角形含有1,2頂點,則從O經過1,2點構成的邊形成一條邊(即出度),如下圖

根據握手定理可知,度之和必爲偶數, 在V1和V2構成的邊上,觀察可得出度必爲奇數,即邊上(1,2).(2,1)的線段必有奇數個,那麼在小三角形必存在奇數度的三角形,即必存在1度的三角形,得證。

 

 

 

 

 

 

 

 

 

 

 

 

 

 

 

 

發表評論
所有評論
還沒有人評論,想成為第一個評論的人麼? 請在上方評論欄輸入並且點擊發布.
相關文章